• Предмет: Геометрия
  • Автор: ansleyyminor
  • Вопрос задан 2 года назад

пожалуйста помогите пронос

Приложения:

Ответы

Ответ дал: yonkada
0

Ответ: да, является, так как соответственно углы при секущей UI равны

Ответ дал: lanserr76
0

Ответ:

да, т.к. углы I и A - соответственные отностельно секущей IU, а т. к. они равны, BA || EI.

Вас заинтересует